Re: [obm-l]

2021-04-23 Por tôpico Matheus Secco
Oi, Marcos. Não é difícil verificar que raiz(2) + raiz_cubica(2) é uma raiz
do polinômio x^6 - 6 x^4 - 4 x^3 + 12 x^2 - 24 x - 4. Com isso, pelo
teorema das raízes racionais, se raiz(2) + raiz_cubica(2) fosse racional,
teria que ser um inteiro e é fácil verificar que 2 < raiz(2) +
raiz_cubica(2) < 3.

Abraços

On Fri, Apr 23, 2021 at 4:43 PM Marcos Martinelli 
wrote:

> Opa, pessoal. Pensei nos últimos dias no problema seguinte. Cheguei a uma
> solução um pouco mais genérica, mas me deu trabalho. Gostaria de estudar
> outras abordagens.
>
> Problema) Prove que raiz (2) + raiz_cúbica (2) é irracional.
>
> Na sequência posto um rascunho do que pensei.
>
> Obrigado.
>
> --
> Esta mensagem foi verificada pelo sistema de antivírus e
> acredita-se estar livre de perigo.

-- 
Esta mensagem foi verificada pelo sistema de antiv�rus e
 acredita-se estar livre de perigo.



[obm-l] Re: [obm-l] Re: [obm-l] Re: [obm-l] Sequência Injetiva

2021-02-16 Por tôpico Matheus Secco
Esse problema caiu na Olimpíada Iberoamericana de 2009 que eu participei.
Foi o problema 5 da prova e lá pedia para provar injetividade e
sobrejetividade.

Em qua, 17 de fev de 2021 00:16, Anderson Torres <
torres.anderson...@gmail.com> escreveu:

> Em dom., 14 de fev. de 2021 às 17:20, Claudio Buffara
>  escreveu:
> >
> > Será que essa sequência é sobrejetiva (sobre os racionais positivos)?
> > Porque como a(2^n) = n+1, ela certamente atinge todos os naturais, de
> modo que é ilimitada, superiormente e inferiormente (já que a(2^n + 1) =
> 1/(n+1) ).
> > Mesmo que não seja, seria interessante descobrir que racionais positivos
> ela não atinge.
> > É suficiente provar que todos os racionais entre 0 e 1 são atingidos (no
> caso, pelos termos de ordem ímpar), mas não sei se isso facilita.
> > Vale uma exploração numérica, talvez com uma planilha.
>
>
> Se eu não errei as contas, acredito que sim. Afinal basta reverter a
> fracao continua.
>
> As operacoes parecem ser bem limitadas, contudo nao e necessario muito
> mais que isso para gerar um racional qualquer:
>
> - Função INC: x -> x+1
> - Função REV: x -> 1/x
>
> Talvez haja algum invariante que permita prever que cada operacao esta
> fadada a cair em 1
>
> >
> >
> > Abs,
> > Claudio.
> >
> > Enviado do meu iPhone
> >
> > Em 14 de fev. de 2021, à(s) 13:57, Anderson Torres <
> torres.anderson...@gmail.com> escreveu:
> >
> > 
> >
> >
> > Em sáb., 13 de fev. de 2021 às 17:56, Jeferson Almir <
> jefersonram...@gmail.com> escreveu:
> >>
> >> Amigos, peço ajuda em provar a injetividade dessa sequência que seria
> uma saída para provar a unica ocorrência do racional que aparece nela.
> Estou andando em círculos tentando montar uma possível indução.
> >>
> >>
> >> Dado a sequência a_1 = 1 e a_2n = a_n  + 1 e a_2n+1 = 1/a_2n.
> >>
> >> Prove que para todo racional positivo que ocorre na sequência, ocorre
> uma única vez.
> >>
> >>
> >
> > Acho que e uma boa usar fracao continua aqui.
> >
> > Se a_n = [c0; c1, c2, ..., ck], temos entao a_1 = [1] e
> >
> > a_2n =Â [(1+c0); c1, c2, ..., ck] (chamemos isso de operacao E)
> > a_2n+1 = [0; (1+c0), c1, c2, ..., ck] (chamemos isso de operacao O)
> >
> >
> > A partir disso, acredito que a bijecao fica quase obvia, bastando
> formalizar algumas inducoes marotas.
> >
> > Primeiramente, nenhuma representacao da forma [...,N,1] vai surgir dai a
> partir de a_2. Isso pode ser demonstrado por inducao mesmo: ck=1 somente
> no caso [1], e depois dele a funcao a_n so modifica o comeco da cadeia,
> nunca o final dela.
> >
> > Assim sendo, temos certeza que nao tem como um racional aparecer uma vez
> na forma canonica e outra na forma alternativa. E, por conseguinte, se duas
> fracoes tem comprimentos diferentes, elas devem ser diferentes. E fracoes
> com comprimentos iguais diferem se e somente se pelo menos um dos
> componentes diferir.
> >
> > Agora, a funcao recursiva age de duas formas. Uma delas altera o
> comprimento em 1, e a outra mantém. A que altera, só altera acrescentando
> o 0 na cabeceira. A que não altera, incrementa a cabeceira.
> >
> > Desta forma, é possível gerar de maneira unica qualquer numeroÂ
> racional comecando do 1.
> >
> > - Qualquer fracao de comprimento 1 pode ser gerada simplesmente
> aplicando a operacao E tantas vezes quantas forem necessarias. E tambem
> nao e possivel fazer isso de outra maneira, pois a operacao O aumentara o
> comprimento de maneira irreversivel.
> >
> > - Dada uma fracao com comprimento K, temos duas sub inducoes para fazer:
> >
> > + A fracao tem comprimento K e comeca com 0.
> >
> > Â  Entao ela foi gerada por uma operacao O. O elemento que a gerou tinha
> menos componentes, os quais satisfazem a hipotese de inducao.
> >
> > + A fracao tem comprimento K e comeca com algo maior que 0.
> >
> > Entao ela foi gerada por uma operacao E. A fracao da qual ela foi gerada
> difere unicamente no primeiro elemento, o qual antes era menor. Assim
> sendo, e possivel reduzir isso ate chegar no caso anterior.
> >
> > E isso demonstra recursivamente a unicidade e existencia!
> >
> >
> >
> >> --
> >> Esta mensagem foi verificada pelo sistema de antivírus e
> >> acredita-se estar livre de perigo.
> >
> >
> > --
> > Esta mensagem foi verificada pelo sistema de antivírus e
> > acredita-se estar livre de perigo.
> >
> >
> > --
> > Esta mensagem foi verificada pelo sistema de antivírus e
> > acredita-se estar livre de perigo.
>
> --
> Esta mensagem foi verificada pelo sistema de antivírus e
>  acredita-se estar livre de perigo.
>
>
> =
> Instru�ões para entrar na lista, sair da lista e usar a lista em
> http://www.mat.puc-rio.br/~obmlistas/obm-l.html
> =
>

-- 
Esta mensagem foi verificada pelo sistema de antiv�rus e
 acredita-se estar livre de perigo.



[obm-l] Re: [obm-l] Resto da divisão de um polinômio

2020-08-22 Por tôpico Matheus Secco
Neste caso específico, você pode usar congruência de polinômios (que é bem
similar à congruência para números inteiros) e isso é facilitado pelo fato
de x^3 - 1 = (x - 1)(x^2+x+1).

Com essa observação, podemos escrever x^3 == 1 (mod x^2+x+1). Com isso,
x^30 = (x^3)^10 == 1 (mod x^2+x+1), x^28 = (x^3)^9 * x == x (mod x^2+x+1) e
7x^12 = 7(x^3)^4 == 7 (mod x^2+x+1).

Assim x^30 - x^28 + 7x^12 == 1 - x + 7 == 8 - x (mod x^2+x+1) e como o grau
de (8-x) é menor que o grau de (x^2+x+1), o resto é 8 - x.

Abraços,

Matheus.

On Sat, Aug 22, 2020 at 9:19 PM Professor Vanderlei Nemitz <
vanderma...@gmail.com> wrote:

> Oi!
>
> Existe algum fato específico que ajude a determinar o resto da divisão de
> um polinômio de grau elevado por outro, ou depende do caso?
>
> Por exemplo, como encontrar o seguinte resto, sem excessivos cálculos?
> Muito obrigado!
>
> *Determine o resto da divisão do polinômio x^30 - x^28 + 7x^12 por x^2 +
> x + 1?*
>
>
> 
>  Livre
> de vírus. www.avast.com
> .
> <#m_2794148347715460488_DAB4FAD8-2DD7-40BB-A1B8-4E2AA1F9FDF2>
>
> --
> Esta mensagem foi verificada pelo sistema de antivírus e
> acredita-se estar livre de perigo.

-- 
Esta mensagem foi verificada pelo sistema de antiv�rus e
 acredita-se estar livre de perigo.



[obm-l] Re: [obm-l] polinômio irredutível

2020-08-16 Por tôpico Matheus Secco
O melhor jeito é pensar na contrapositiva (supondo que você esteja falando
sobre irredutibilidade em Z[x] ou até em Q[x]): se f(x) fatora como
g(x)*h(x), então f(x+a) fatora como g(x+a) *h(x+a) e é claro que uma vez
que g(x) e h(x) têm coeficientes inteiros, então g(x+a) e h(x+a) também
têm. A recíproca é essencialmente idêntica.

Abraços

Em dom, 16 de ago de 2020 14:11, Luís Lopes 
escreveu:

> Sauda,c~oes,
>
> Como provar que um polinômio f(x) tendo como coeficientes números inteiros
> é irredutível se e somente se f(x+a) é irredutível para algum  inteiro
> ?
>
> Luís
>
>
>
>
> --
> Esta mensagem foi verificada pelo sistema de antivírus e
> acredita-se estar livre de perigo.
>

-- 
Esta mensagem foi verificada pelo sistema de antiv�rus e
 acredita-se estar livre de perigo.



[obm-l] Re: [obm-l] Geometria plana com desigualdade de médias?

2020-08-16 Por tôpico Matheus Secco
Olá, Vanderlei.
Por Cauchy-Schwarz, temos

(a/ha + b/hb + c/hc) * (a*ha + b*hb + c*hc) >= (a+b+c)^2.  (#)

Como (a*ha + b*hb + c*hc) = 2S, onde S é a área de ABC, segue que a
expressão a/ha + b/hb + c/hc é pelo menos 2p^2/S, onde p é o
semi-perimetro.

Por outro lado, a igualdade em (#) ocorre se, e somente se, ha = hb = hc,
ou seja, quando P é o incentro do triângulo

Abraços,
Matheus

Em dom, 16 de ago de 2020 08:59, Professor Vanderlei Nemitz <
vanderma...@gmail.com> escreveu:

> Bom dia!
>
> Tentei utilizar alguma desigualdade de médias aqui, mas não tive êxito.
> Alguém ajuda?
> Muito agradecido!
>
> Seja P um ponto no interior de um triângulo e sejam ha, hb e hc as
> distâncias de P aos lados a, b e c, respectivamente. Mostre que o valor
> mínimo de (a/ha) + (b/hb) + (c/hc) ocorre quando P é o incentivo do
> triângulo ABC.
>
> --
> Esta mensagem foi verificada pelo sistema de antivírus e
> acredita-se estar livre de perigo.

-- 
Esta mensagem foi verificada pelo sistema de antiv�rus e
 acredita-se estar livre de perigo.



Re: [obm-l]

2020-02-15 Por tôpico Matheus Secco
Em sex, 14 de fev de 2020 19:49, Luís Lopes 
escreveu:

> Minhas mensagens não estão chegando. Tento mais uma vez.
>
> Sauda,c~oes,
>
> Construir o triângulo (sinteticamente, sem (muita) álgebra)
> com os dados acima. k é um número real (construtível) conhecido.
>
>  Não sei se pode servir como aquecimento mas o problema
>  me parece mais fácil.
>
> Fonte: Il Problema Geometrico Dal compasso al Cabri.
> Italo D'Ignazio e Ercole Supra.
>
> O segundo aparece no Petersen também.
>
> Os problemas  e  são casos particulares com k=1.
>
> Abraços,
> Luís
>
>
> --
> Esta mensagem foi verificada pelo sistema de antivírus e
> acredita-se estar livre de perigo.
>

-- 
Esta mensagem foi verificada pelo sistema de antiv�rus e
 acredita-se estar livre de perigo.



Re: [obm-l] Determinante

2019-06-05 Por tôpico Matheus Secco
Oi, Ralph, acho que você quis dizer trocar a linha 3 por essa combinação
linear que colocou.
Você só pode trocar uma linha por ela mais uma combinação linear das
*outras*, certo?

Abraços

Em qua, 5 de jun de 2019 22:20, Ralph Teixeira  escreveu:

> As propriedades importantes aqui sao:
>
> -- O determinante nao muda se voce trocar uma linha (ou coluna) por uma
> combinacao linear dela com as outras;
> -- O determinante eh linear em CADA linha (ou coluna); em particular, se
> uma linha eh divisivel por 13, voce pode "fatorar" este 13 desta linha para
> fora do determinante.
>
> Entao, experimente trocar a linha L1 por 100*L1+10*L2+L3... Agora use o
> que voce tinha visto para "tirar" o 13 da primeira linha, e o que sobra eh
> claramente um inteiro.
>
> Abraco, Ralph.
>
> On Wed, Jun 5, 2019 at 9:49 PM Daniel da Silva <
> danielrochadasi...@icloud.com> wrote:
>
>> Boa noite pessoal,
>>
>> Não estou conseguindo um argumento para essa questão:
>>
>> Mostrar sem desenvolver que o determinate de:
>> 1  2  5
>> 6  7  4
>> 9  3  6
>>
>> É divisível por 13.
>>
>> Reparei que 169, 273, 546 são divisíveis por 13, mas não consegui
>> pensar em nada para usar isso.
>>
>> Obrigado,
>> Daniel
>> --
>> Esta mensagem foi verificada pelo sistema de antivírus e
>>  acredita-se estar livre de perigo.
>>
>>
>> =
>> Instruções para entrar na lista, sair da lista e usar a lista em
>> http://www.mat.puc-rio.br/~obmlistas/obm-l.html
>> =
>>
>
> --
> Esta mensagem foi verificada pelo sistema de antivírus e
> acredita-se estar livre de perigo.

-- 
Esta mensagem foi verificada pelo sistema de antiv�rus e
 acredita-se estar livre de perigo.



Re: [obm-l] algebra

2019-02-15 Por tôpico Matheus Secco
Oi, Ralph, aproveitando a sua ideia, veja que ele pede abc-1 e
multiplicando as suas equações, você tira abc rapidinho.

Abraços

Em sáb, 16 de fev de 2019 01:26, Ralph Teixeira  Tome a=x+1, b=y+1 e c=z+1.
>
> As equacoes equivalem a:
>
> ab=9
> bc=16
> ac=36
>
> que nao sao dificeis de resolver -- multiplique duas delas, divida pela
> outra, use que a,b,c>0 Fica a=9/2; b=2; c=8.
>
> Entao x=7/2; y=1 e z=7, e daqui voce tira o que precisar.
>
> Abraco, Ralph.
>
>
>
>
> On Fri, Feb 15, 2019 at 7:54 PM marcone augusto araújo borges <
> marconeborge...@hotmail.com> wrote:
>
>> assuma que x, y, z são numeros positivos tais que satisfazem as equações
>> abaixo . Determine o valor de xyz + x+y+z
>>
>> x+y+xy = 8
>> y+z+yz = 15
>> z+x+ zx = 35
>>
>> Eu encontrei  xyz + x+y+z + xy +xz + yz = 71, mas...
>> o gabarito diz que a resposta é 36
>>
>> --
>> Esta mensagem foi verificada pelo sistema de antivírus e
>> acredita-se estar livre de perigo.
>>
>
> --
> Esta mensagem foi verificada pelo sistema de antivírus e
> acredita-se estar livre de perigo.

-- 
Esta mensagem foi verificada pelo sistema de antiv�rus e
 acredita-se estar livre de perigo.



[obm-l] Re: [obm-l] Re: [obm-l] Re: [obm-l] Equações do 2 grau

2018-08-20 Por tôpico Matheus Secco
Na verdade, foi construída essa função auxiliar para reinterpretar os dados
do problema de outra maneira que fosse útil.

Em seg, 20 de ago de 2018 11:01, Alexandre Antunes <
prof.alexandreantu...@gmail.com> escreveu:

> Bom dia,
>
> Mas o enunciado de 1a não estaria incompleto?
> - o que diz que a expressão é relativa a uma equação (ou função) do 2°
> grau?
> - E se a função suposta for outra?
>
> Em Seg, 20 de ago de 2018 10:09, Matheus Secco 
> escreveu:
>
>> Para a primeira, supondo a, b, c reais, considere a função quadrática
>> f(x) = cx² + bx + a e veja que a^2+ab+ac = a(a+b+c) = f(0) * f(1).
>> Do enunciado, tem-se f(0) * f(1) < 0 e isso significa que a função possui
>> exatamente 1 raiz entre 0 e 1. Por se tratar de uma função quadrática, deve
>> ter outra raiz real, que está fora do intervalo (0,1). Com isso, possui
>> duas raízes reais distintas e então o discriminante b² - 4ac é positivo:
>> b²> 4ac.
>>
>> On Sun, Aug 19, 2018 at 6:21 PM Daniel Quevedo 
>> wrote:
>>
>>> 1) Se a^2 +ab + ac < 0, então:
>>> A) a^2 > 4ab
>>> B) b^2 > 4ac
>>> C) c^2 > 4ab
>>> D) a^2 = 4b
>>> E) b^2 = 4ac
>>>
>>> R: B
>>>
>>> 2) sendo a, b e c inteiros ímpares, sobre as raizes da equação ax^2 + bx
>>> + c = 0 podemos afirmar que:
>>> A) são inteiros ímpares
>>> B) são inteiros pares
>>> C) não são racionais
>>> D) são racionais não inteiras
>>> E) não são reais
>>>
>>> R: C
>>> --
>>> Fiscal: Daniel Quevedo
>>>
>>> --
>>> Esta mensagem foi verificada pelo sistema de antivírus e
>>> acredita-se estar livre de perigo.
>>
>>
>> --
>> Esta mensagem foi verificada pelo sistema de antivírus e
>> acredita-se estar livre de perigo.
>
>
> --
> Esta mensagem foi verificada pelo sistema de antivírus e
> acredita-se estar livre de perigo.

-- 
Esta mensagem foi verificada pelo sistema de antiv�rus e
 acredita-se estar livre de perigo.



[obm-l] Re: [obm-l] Equações do 2 grau

2018-08-20 Por tôpico Matheus Secco
Para a primeira, supondo a, b, c reais, considere a função quadrática f(x)
= cx² + bx + a e veja que a^2+ab+ac = a(a+b+c) = f(0) * f(1).
Do enunciado, tem-se f(0) * f(1) < 0 e isso significa que a função possui
exatamente 1 raiz entre 0 e 1. Por se tratar de uma função quadrática, deve
ter outra raiz real, que está fora do intervalo (0,1). Com isso, possui
duas raízes reais distintas e então o discriminante b² - 4ac é positivo:
b²> 4ac.

On Sun, Aug 19, 2018 at 6:21 PM Daniel Quevedo  wrote:

> 1) Se a^2 +ab + ac < 0, então:
> A) a^2 > 4ab
> B) b^2 > 4ac
> C) c^2 > 4ab
> D) a^2 = 4b
> E) b^2 = 4ac
>
> R: B
>
> 2) sendo a, b e c inteiros ímpares, sobre as raizes da equação ax^2 + bx +
> c = 0 podemos afirmar que:
> A) são inteiros ímpares
> B) são inteiros pares
> C) não são racionais
> D) são racionais não inteiras
> E) não são reais
>
> R: C
> --
> Fiscal: Daniel Quevedo
>
> --
> Esta mensagem foi verificada pelo sistema de antivírus e
> acredita-se estar livre de perigo.

-- 
Esta mensagem foi verificada pelo sistema de antiv�rus e
 acredita-se estar livre de perigo.



[obm-l] Re: [obm-l] Polinômio com raízes reais

2018-07-04 Por tôpico Matheus Secco
Se o polinômio tiver apenas raízes simples, isto é consequência do Teorema
de Rolle.

Caso haja alguma raiz com multiplicidade k, pelo menos 2, basta usar que a
raiz anula também as derivadas de ordem até k - 1.

Abraços,

Matheus Secco

On Wed, Jul 4, 2018 at 11:27 PM Artur Steiner 
wrote:

> Acho um tanto surpreendente que este fato não pareça ser muito conhecido:
>
> Se todas as raízes de um polinômio P de grau >= 2 forem reais, então todas
> as raízes de P' também são.
>
> Isso vale inclusive para polinômios complexos. Mas basta provar para
> polinômios com coeficientes reais.
>
>
> Artur Costa Steiner
>
> --
> Esta mensagem foi verificada pelo sistema de antivírus e
> acredita-se estar livre de perigo.

-- 
Esta mensagem foi verificada pelo sistema de antiv�rus e
 acredita-se estar livre de perigo.



[obm-l] Re: [obm-l] Re: [obm-l] Produto das distâncias máximo

2018-05-22 Por tôpico Matheus Secco
Completando o trabalho do Claudio, não é dificil mostrar que P deve então
ser o baricentro.

Em Ter, 22 de mai de 2018 10:37, Claudio Buffara 
escreveu:

> Sejam x, y, z as distâncias do ponto P, interior ao triângulo ABC, de área
> S, aos lados BC (medida = a), AC (medida = b) e AB (medida = c).
>
> Sabemos que ax + by + cz = 2S = constante.
>
> Então, o problema é maximizar xyz dado que ax + by + cz = 2S.
>
> Pela desigualdade MG <= MA aplicada aos números positivos ax, by e cz,
> temos que:
> ax * by * cz <= ((ax + by + cz)/3)^3 ==>
> abc*xyz <= (2S/3)^3 ==>
> xyz <= (2S/3)^3/(abc), com igualdade sss ax = by = cz = 2S/3.
>
> Assim, o ponto P que maximiza o produto xyz é tal que as áreas dos
> triângulos PAB, PBC e PCA são iguais.
>
> []s,
> Claudio.
>
>
> 2018-05-22 8:39 GMT-03:00 luciano rodrigues :
>
>> Encontre o ponto dentro de um triângulo tal que o produto das
>> distâncias dos lados desse triângulo ao ponto seja máximo.
>>
>> --
>> Esta mensagem foi verificada pelo sistema de antivírus e
>>  acredita-se estar livre de perigo.
>>
>>
>> =
>> Instruções para entrar na lista, sair da lista e usar a lista em
>> http://www.mat.puc-rio.br/~obmlistas/obm-l.html
>> =
>>
>
>
> --
> Esta mensagem foi verificada pelo sistema de antivírus e
> acredita-se estar livre de perigo.

-- 
Esta mensagem foi verificada pelo sistema de antiv�rus e
 acredita-se estar livre de perigo.



[obm-l] Re: [obm-l] Re: [obm-l] Re: [obm-l] Re: [obm-l] Teoria dos números

2018-03-26 Por tôpico Matheus Secco
De fato, trata-se do problema 1 da IMO 1992.

Abs,

Matheus Secco

Em Seg, 26 de mar de 2018 09:24, Claudio Buffara <claudio.buff...@gmail.com>
escreveu:

> Muito fácil pra ser de IMO...
>
> 2018-03-26 6:58 GMT-03:00 Anderson Torres <torres.anderson...@gmail.com>:
>
>> Este não é o problema de alguma IMO não? Eu lembro de ter resolvido,
>> quase igual à solução oficial: substituir s,t,u por a+1,b+1,c+1 e
>> calcular os possiveis valores de
>> 1/a+1/b+1/c + 1/ab+1/ac+1/bc usando desigualdades - para daí limitar
>> os valores de a,b,c.
>>
>> Em 23 de março de 2018 17:01, Claudio Buffara
>> <claudio.buff...@gmail.com> escreveu:
>> > Enfim, nesse meio tempo acho que resolvi o problema...
>> >
>> > Devemos achar inteiros s, t, u, com 1 < s < t < u e tais que:
>> > (stu -1)/((s-1)(t-1)(u-1)) = k  (k inteiro positivo)
>> >
>> > Após diversas aplicações do truque (método?) de somar e subtrair a mesma
>> > coisa, chegamos a:
>> > stu - 1 =  (s-1)(t-1)(u-1) + (s-1)(t-1) + (s-1)(u-1) + (t-1)(u-1) +
>> (s-1) +
>> > (t-1) + (u-1)
>> >
>> > Dividindo isso por (s-1)(t-1)(u-1), obtemos:
>> > 1 + 1/(u-1) + 1/(t-1) + 1/(s-1) + 1/((t-1)(u-1)) + 1/((s-1)(u-1)) +
>> > 1/((s-1)(t-1)) = k ==>
>> >
>> > 1/(u-1) + 1/(t-1) + 1/(s-1) + 1/((t-1)(u-1)) + 1/((s-1)(u-1)) +
>> > 1/((s-1)(t-1)) = k-1
>> >
>> > Agora a ideia é achar cotas para s e para k.
>> >
>> > 1 < s < t < u ==> s >= 2, t >= 3 e u >= 4 ==> o lado esquerdo é menor ou
>> > igual que:
>> > 1/3 + 1/2 + 1 + 1/6 + 1/3 + 1/2 = 2+5/6
>> >
>> > Ou seja, como o lado esquerdo é inteiro (e positivo), só poderá ser
>> igual a
>> > 1 ou a 2 ==> k = 2 ou k = 3.
>> >
>> > Se s >= 4, então t >= 5 e u >= 6, e o lado esquerdo será, no máximo,
>> igual
>> > a:
>> > 1/5 + 1/4 + 1/3 + 1/20 + 1/15 + 1/12 < 1.
>> >
>> > Logo, devemos ter s = 2 ou s = 3.
>> >
>> > s = 2 ==>
>> > 1/(u-1) + 1/(t-1) + 1 + 1/((t-1)(u-1)) + 1/(u-1) + 1/(t-1) = k-1 ==>
>> > 2/(t-1) + 2/(u-1) + 1/((t-1)(u-1)) = k-2 ==>
>> > Como k-2 deve ser inteiro positivo, k só pode ser 3 e, portanto:
>> > 2/(t-1) + 2/(u-1) + 1/((t-1)(u-1)) = 1 ==>
>> > (2 + 1/(t-1))/(u-1) = 1 - 2/(t-1) ==>
>> > u = 1 + (2t - 1)/(t - 3) = 3 + 5/(t-3) ==>
>> > t = 4 e u = 8   ou   t = 8 e u = 4 (não serve pois t deve ser menor do
>> que
>> > u)
>> >
>> > s = 3 ==>
>> > 1/(u-1) + 1/(t-1) + 1/2 + 1/((t-1)(u-1)) + 1/(2(u-1)) + 1/(2(t-1)) =
>> k-1 ==>
>> > (3/2)/(u-1) + (3/2)/(t-1) + 1/((t-1)(u-1)) = k - 3/2 ==>
>> > 3/(u-1) + 3/(t-1) + 2/((t-1)(u-1)) = 2k - 3 ==>
>> > (3 + 2/(t-1))/(u-1) = 2k - 3t/(t-1) ==>
>> > (3t - 1)/(u-1) = 2k(t-1) - 3t ==>
>> > u = 1 + (3t - 1)/((2k-3)t - 2k)
>> >
>> > k = 2 ==> u = 1 + (3t-1)/(t-4) = 4 + 11/(t-4) ==> t = 5 e u = 15
>> >
>> > k = 3 ==> u = 1 + (3t-1)/(3t-6) = 2 + 5/(3t-6) ==> XXX
>> >
>> > As únicas soluções são:
>> > (2,4,8) e (3,5,15)
>> >
>> > []s,
>> > Claudio.
>> >
>> > 2018-03-23 15:38 GMT-03:00 Pedro José <petroc...@gmail.com>:
>> >>
>> >> Boa tarde!
>> >>
>> >> Aproveitando que deu o que falar o problema postado pelo Douglas, tem
>> um
>> >> que achei mais interessante.
>> >>
>> >> (s-1)(t-1).(u-1) | stu -1, com s, t, u inteiros  e 1 <s<t> >>
>> >> Saudações,
>> >> Pedro
>> >>
>> >> --
>> >> Esta mensagem foi verificada pelo sistema de antivírus e
>> >> acredita-se estar livre de perigo.
>> >
>> >
>> >
>> > --
>> > Esta mensagem foi verificada pelo sistema de antivírus e
>> > acredita-se estar livre de perigo.
>>
>> --
>> Esta mensagem foi verificada pelo sistema de antivírus e
>>  acredita-se estar livre de perigo.
>>
>>
>> =
>> Instru�ões para entrar na lista, sair da lista e usar a lista em
>> http://www.mat.puc-rio.br/~obmlistas/obm-l.html
>> =
>>
>
>
> --
> Esta mensagem foi verificada pelo sistema de antivírus e
> acredita-se estar livre de perigo.

-- 
Esta mensagem foi verificada pelo sistema de antiv�rus e
 acredita-se estar livre de perigo.



[obm-l] Re: [obm-l] Álgebra

2018-03-13 Por tôpico Matheus Secco
Olá Douglas, use que
(x+y+z)^5 - x^5 - y^5 - z^5 = 5(x+y)(y+z)(z+x)(x^2+y^2+z^2+xy+yz+zx),
tomando x = a - b + c, y = a + b - c e z = b + c - a.
Isso te dará 80abc(a²+b²+c²).

Abraços

2018-03-13 18:51 GMT-03:00 Douglas Oliveira de Lima <
profdouglaso.del...@gmail.com>:

> Olá meus amigos, vocês conhecem um jeito bom de simplificar isso
> (a+b+c)^5-(a-b+c)^5-(a+b-c)^5-(b+c-a)^5
>
> Abraços
> Douglas Oliveira
>
> --
> Esta mensagem foi verificada pelo sistema de antivírus e
> acredita-se estar livre de perigo.

-- 
Esta mensagem foi verificada pelo sistema de antiv�rus e
 acredita-se estar livre de perigo.



[obm-l] Re: [obm-l] Re: [obm-l] Re: [obm-l] polinômios

2017-11-28 Por tôpico Matheus Secco
Para ver que Q(x), basta ver que (x-a)(x-b)(x-c)(x-d) tem coeficiente lider
1 e ao fazer a divisão longa de P(x) por este polinomio com coeficiente
lider 1, não há riscos de introduzir frações.

Abs,
Secco

Em 28 de nov de 2017 11:58 AM, "Carlos Nehab" 
escreveu:

Oi, Ralph

E o detalhe que Q(x) tem coeficientes inteiros..., "exprica prá nóis"!

Abraços
Nehab

Em 27 de novembro de 2017 21:51, Ralph Teixeira 
escreveu:

> Acho que eles queriam 4 raizes inteiras distintas.
>
> Neste caso, temos P(x)=(x-a)(x-b)(x-c)(x-d)Q(x) onde Q(x) tem coeficientes
> inteiros e a,b,c,d sao as 4 raizes inteiras distintas.
>
> Se P(x)=2 tivesse raiz inteira, digamos, x=n, entao teriamos
> P(n)=(n-a)(n-b)(n-c)(n-d)Q(n)=2. Mas entao n-a, n-b, n-c e n-d seriam 4
> inteiros distintos cujo produto seria +-1 ou +-2, o que nao eh possivel.
>
> Abraco, Ralph.
>
> 2017-11-27 20:09 GMT-02:00 André Lauer :
>
>> Boa noite, preciso de ajuda no seguinte problema:
>> Um polinômio P(x) tem coeficientes inteiros e admite quatro raízes
>> inteiras. Prove que a equação P(x) = 2 não admite raízes inteiras.
>>
>> --
>> Esta mensagem foi verificada pelo sistema de antivírus e
>> acredita-se estar livre de perigo.
>>
>
>
> --
> Esta mensagem foi verificada pelo sistema de antivírus e
> acredita-se estar livre de perigo.



-- 
Esta mensagem foi verificada pelo sistema de antivírus e
acredita-se estar livre de perigo.

-- 
Esta mensagem foi verificada pelo sistema de antiv�rus e
 acredita-se estar livre de perigo.



Re: [obm-l] Livro de Topologia

2017-09-26 Por tôpico Matheus Secco
Eu recomendo o do James Munkres.

Em ter, 26 de set de 2017 às 09:04, Luiz Antonio Rodrigues <
rodrigue...@gmail.com> escreveu:

> Olá, pessoal!
> Bom dia!
> Dei uma olhada na Amazon e vi muitos títulos de Topologia bem avaliados.
> São tantos que eu fiquei perdido...
> Alguém conhece um bom título?
> Muito obrigado e um abraço!
> Luiz
>
> --
> Esta mensagem foi verificada pelo sistema de antivírus e
> acredita-se estar livre de perigo.

-- 
Esta mensagem foi verificada pelo sistema de antiv�rus e
 acredita-se estar livre de perigo.



Re: [obm-l] Problema de grafos

2017-09-02 Por tôpico Matheus Secco
Olá Daniel, veja que os graus podem variar de 0 até n - 1. Entretanto, não
é possível ter um vértice com grau 0 e outro com grau n - 1. Desta forma,
em vez de n possibilidades para o grau de cada vértice, há n - 1
possibilidades para o grau de cada vértice. Como há n vértices, pelo
Princípio da Casa dos Pombos, há dois com o mesmo grau.
Abraços,
Matheus Secco

2017-09-02 11:26 GMT-03:00 Daniel Rocha <danielrochadasi...@icloud.com>:

> Bom dia,
>
> Seja G um grafo com n vértices, n maior que 1. Suponha que G não possua
> loops nem mais de uma aresta unindo pares de vértices. Prove que G possui
> dois vértices de graus iguais.
>
> Obrigado,
> Daniel
> --
> Esta mensagem foi verificada pelo sistema de antivírus e
>  acredita-se estar livre de perigo.
>
>
> =
> Instruções para entrar na lista, sair da lista e usar a lista em
> http://www.mat.puc-rio.br/~obmlistas/obm-l.html
> =
>

-- 
Esta mensagem foi verificada pelo sistema de antiv�rus e
 acredita-se estar livre de perigo.



Re: [obm-l] Problema estranho

2017-07-11 Por tôpico Matheus Secco
Oi Ralph, tava sem tempo de escrever, mas vou aproveitar a deixa porque
você já fez quase tudo. Acho que dá pra fazer o caso geral usando que os
reais admitem uma base considerando como um espaço vetorial sobre os
racionais.
Em ter, 11 de jul de 2017 às 18:18, Ralph Teixeira 
escreveu:

> Bom, eu sei resolver se todos os números forem racionais. Deve ter um
> jeito de usar isso para o caso geral...
>
> A propriedade desse conjunto não se altera se todos os elementos do
> conjunto forem multiplicados por um mesmo número, nem se a gente somar uma
> certa constante a todos eles.
>
> Assim, *SE* eles forem todos racionais, a gente pode multiplicar todos
> eles por um m.m.c imenso e supor que são todos inteiros, spdg.
>
> Mas então todos teriam que ter a mesma paridade -- afinal a soma de todos
> eles, menos qualquer um deles, é um número par.
>
> Então, enquanto todos forem pares, divida-os por 2; em algum momento,
> **todos** ficarão ímpares. Quando isso acontecer, some 1, e ficam todos
> pares. Então divida por 2 de novo, e de novo, até ficarem ímpares, então
> some 1 de novo, repita e enxágue.
>
> Esse processo vai parar? Oras, esses inteiros vão diminuir em módulo
> até até até cada um deles virar 0, ou 1! De fato, |x|/2<|x| quando
> x<>0, e |x+1|/2 < |x| para x<>0,1. Então a cada um ou dois passos o valor
> absoluto de todos eles diminui -- a menos que eles sejam 0 ou 1. Ou seja,
> em tempo finito, todos eles vão virar 0 ou 1.
>
> Agora é fácil -- lembra que todos sempre têm a mesma paridade?? Então são
> todos 0, ou todos 1.
>
> ---///---
>
> Para o caso geral, tenho uma ideia, mas não estou com tempo de
> desenvolvê-la -- será que dá para começar com os reais, e multiplicar todos
> eles por algum número real imenso de forma que eles sejam quase inteiros
> (tipo, todos eles a menos de 1/(4n) de algum inteiro)? Talvez dê para
> mostrar então pela propriedade que eles têm que ser inteiros, ou pelo menos
> "comensuráveis" e daí matar o problema.
>
> Abraço, Ralph.
>
>
>
> 2017-07-11 15:06 GMT-03:00 Nowras Ali :
>
>> Uma prova por indução me parece o melhor caminho.
>> O Bernardo já provou para o caso base, basta agora tentar
>> provar para n+1, assumindo verdadeiro para n. Tentarei resolver
>> o problema assim que puder.
>>
>> Abraços, Nowras.
>>
>> Em 9 de julho de 2017 18:54, Otávio Araújo 
>> escreveu:
>>
>>>
>>> Já tentei isso, porém não parece ajudar em muita coisa  mas de
>>> qualquer forma obrigado
>>>
>>> > Em 9 de jul de 2017, às 18:00, Bernardo Freitas Paulo da Costa <
>>> bernardo...@gmail.com> escreveu:
>>> >
>>> > Não pensei muito, mas acho que você deveria tentar provar os casos
>>> n=1
>>> > e n=2 "no braço" para ter a intuição.  E, na verdade, o enunciado
>>> > deveria ser: dados a_1, a_2, ... a_{2n+1} números reais, não
>>> > necessariamente distintos, tais que, para cada escolha de 2n dentre
>>> > eles, é possível separar em dois grupos de n cada, com a mesma soma.
>>> > (evitando falar de conjuntos, você pode ter à vontade os elementos
>>> > repetidos).
>>> >
>>> > Assim, o caso n=1 fica: temos a_1, a_2, a_3.  Tomando os elementos
>>> > a_1, a_2, é possĩvel separar em dois grupos de um elemento, com a
>>> soma
>>> > igual.  Logo a_1 = a_2.  Por simetria, a_1 = a_3, e acabou.  Para n=2,
>>> > dá mais trabalho.
>>> >
>>> > 2017-07-08 23:20 GMT+03:00 Otávio Araújo >> >:
>>> >> Galera, queria que alguém pudesse resolver essa questão pra mim
>>> ( passei muito tempo nela já kkk):
>>> >> " Seja n um natural positivo e A um conjunto de 2n+1 números
>>> reais, não necessariamente distintos, com a seguinte propriedade:
>>> >> - Todo subconjunto de A com 2n elementos pode ser particionado em
>>> dois conjuntos de n elementos tais que a soma dos elementos de cada um
>>> desses dois conjuntos de n elementos são iguais.
>>> >>   Prove que todos os elementos de A são iguais."
>>> >>
>>> >>
>>> >>
>>> >>
>>> >>
>>> >>
>>> >>
>>> >> --
>>> >> Esta mensagem foi verificada pelo sistema de antivírus e
>>> >> acredita-se estar livre de perigo.
>>> >>
>>> >>
>>> >>
>>> =
>>> >> Instruções para entrar na lista, sair da lista e usar a lista em
>>> >> http://www.mat.puc-rio.br/~obmlistas/obm-l.html
>>> >>
>>> =
>>> >
>>> >
>>> >
>>> > --
>>> > Bernardo Freitas Paulo da Costa
>>> >
>>> > --
>>> > Esta mensagem foi verificada pelo sistema de antivírus e
>>> > acredita-se estar livre de perigo.
>>> >
>>> >
>>> >
>>> =
>>> > Instruções para entrar na lista, sair da lista e usar a lista em
>>> > http://www.mat.puc-rio.br/~obmlistas/obm-l.html
>>> >
>>> =
>>>
>>> --
>>> Esta mensagem foi 

Re: [obm-l] Re: [obm-l] Re: [obm-l] Re: [obm-l] Duas questões de matemática.

2016-08-10 Por tôpico Matheus Secco
Boa noite!
De acordo com o Fundamentos da Matematica Elementar, a definição de ângulos 
suplementares é apenas para dois ângulos. 

Enviado do meu iPhone

> Em 10 de ago de 2016, às 19:28, Leandro Martins  
> escreveu:
> 
> Olá, amigos!
> 
> Quanto à questão filosófica: sabe-se que a soma  dos ângulos internos de 
> um triângulo, na geometria euclidiana plana, resulta 180 graus. Mas tais 
> ângulos não são definidos como suplementares.
> 
> Teríamos, aqui, uma pista de resposta negativa à questão de Douglas?
> 
> Abraço,
> 
> Leandro
> 
> 
> Em 10/08/2016 13:50, "Pedro José"  escreveu:
>> Boa tarde!
>> 
>> Peguei um livro antigo do ginásio e a definição que lá consta é para 
>> dois ângulos.
>> Mas como as coisas mudam. Pesquisei em sítios do Brasil, EUA e França, 
>> todas as definições são para dois ângulos.
>> Já que se está falando em definições, quando estudava Análise no 
>> científico, Z+  incluía o 0 e Z+*  não incluía o 0, se chamavam, 
>> inteiros positivos e inteiros estritamente positivos (o asterisco 
>> simbolizava a exclusão do 0). 
>> Atualmente Z+ já exclui o 0. 
>> Alguém saberia dizer, quando e o porquê da mudança?
>> 
>> Saudações,
>> PJMSÂ Â Â Â Â Â Â Â Â Â 
>> 
>> Em 8 de agosto de 2016 18:53, Rogerio Ponce  escreveu:
>>> Ola' Douglas, a questao me parece perfeita.
>>> Como as opcoes de resposta sao positivas, queremos a menor quantidade de 
>>> derrotas (ou seja, a maior quantidade de vitorias), que leve ao mesmo total 
>>> de premios.
>>> Portanto, estamos falando das derrotas de maior valor (foram as 4 ultimas), 
>>> acompanhadas por uma com o valor necessario para completar a soma total.
>>> Se considerassemos as 5 ultimas derrotas, o valor total seria ultrapassado.
>>> So' pode ser a letra "E".
>>> []'s
>>> Rogerio Ponce
>>> 
>>> 2016-08-08 16:45 GMT-03:00 Douglas Oliveira de Lima 
>>> :
 Olá amigos, gostaria de uma ajuda em uma filosofia e uma questão.
 
 1)Na definição de ângulos suplementares, seria para dois ângulos ou 
 pode ser para mais de dois?
 
 2)(Essa questão gostaria de saber se está mal elaborada) Carlos e 
 Ricardo disputaram 15 partidas de boliche e ao fim de cada partida o 
 perdedor pagava um prêmio em dinheiro para o vencedor. O prêmio para a 
 primeira partida foi R$ 15,00 e o prê- mio de cada partida seguinte foi 
 R$ 5,00 a mais do que o valor da partida anterior. Ao final da disputa, 
 ambos receberam o mesmo valor em dinheiro e nenhuma partida terminou 
 empatada. Nes- sas condições, a maior diferença possível entre as 
 vitórias e as derrotas de Ricardo é
 (A) 4. (B) 3. (C) 7. (D) 6. (E) 5.
 
 Att: Douglas Oliveira.
 
 -- 
 Esta mensagem foi verificada pelo sistema de antivírus e 
 acredita-se estar livre de perigo.
>>> 
>>> 
>>> -- 
>>> Esta mensagem foi verificada pelo sistema de antivírus e 
>>> acredita-se estar livre de perigo.
>> 
>> 
>> -- 
>> Esta mensagem foi verificada pelo sistema de antivírus e 
>> acredita-se estar livre de perigo.
> 
> -- 
> Esta mensagem foi verificada pelo sistema de antivírus e 
> acredita-se estar livre de perigo.

-- 
Esta mensagem foi verificada pelo sistema de antiv�rus e
 acredita-se estar livre de perigo.



Re: [obm-l] Primo?

2015-11-24 Por tôpico Matheus Secco
Acredito que você possa usar resíduos quadráticos:

(2 legendre p) = (-1)^(p^2-1)/8

(2 legendre p) == 2^(p-1)/2 (mód p)

Para p = 167, temos que (167^2-1)/8 é par. Logo (2 legendre 167) = 1.
Com isso, obtemos que 2^83 == 1 (mód 167).

Abraços

2015-11-24 10:16 GMT-02:00 Pacini Bores :

>
>
>
> Olá Marcone,
>
> Observe que 2^166-1 é divisível por  167; logo   um dos fatores de
> (2^83-1)(2^83+1) divide 167, já que 167 é primo. Só estou tentando provar
> que é 2^83-1, que ainda não consegui.
>
> Pacini
>
> Em 24/11/2015 7:32, marcone augusto araújo borges escreveu:
>
> Mostre que 2^83 - 1 não é primo
>
> --
> Esta mensagem foi verificada pelo sistema de antivírus e
> acredita-se estar livre de perigo.
>
>
> --
> Esta mensagem foi verificada pelo sistema de antivírus e
> acredita-se estar livre de perigo.
>

-- 
Esta mensagem foi verificada pelo sistema de antiv�rus e
 acredita-se estar livre de perigo.



Re: [obm-l] Matriz nxn

2015-11-05 Por tôpico Matheus Secco
Você também pode usar o teorema de Jacobi e trocar a primeira coluna por
ela mais todas as outras.
A primeira coluna passa a ser composta por (x+(n-1)a). Coloca esse cara em
evidência, usa Chió e aí você fica com uma matriz de ordem n-1 diag(x-a,
..., x-a), cujo det é (x-a)^(n-1).

2015-11-04 3:40 GMT-02:00 Marcelo Salhab Brogliato :

> Oi, Eduardo, boa noite.
>
> Essa é uma matrix circular (https://en.wikipedia.org/wiki/Circulant_matrix).
> Assim:
> det(M) = \prod_{j=0}^{n-1} [x + a(w_j + w_j^2 + w_j^3 + ... + w_j^{n-1})]
>
> Onde w_j é a j-ésima raiz unitária, isto é, w_j^n = 1.
>
> Mas, para w_j != 1, temos: w_j + w_j^2 + ... + w_j^{n-1} = w_j (1 -
> w_j^(n-1)) / (1 - w_j) = (w_j - w_j^n) / (1 - w_j) = (w_j - 1) / (1 - w_j)
> = -1. Assim:
> det(M) = [x+(n-1)a] \prod_{j=1}^{n-1} (x - a) = [x+(n-1)a](x-a)^{n-1}
>
> Abraços,
> Salhab
>
>
> 2015-11-03 23:43 GMT-02:00 Anderson Torres :
>
>> Você quer dizer algo assim, por exemplo?
>>
>> X A A A A
>> A X A A A
>> A A X A A
>> A A A X A
>> A A A A X
>>
>>
>> Em 3 de novembro de 2015 23:42, Anderson Torres
>>  escreveu:
>> > Dê um exemplo. Não entendi nada.
>> >
>> > Em 3 de novembro de 2015 22:26, Eduardo Henrique
>> >  escreveu:
>> >> Pessoas, me deparei com a seguinte questão:
>> >>
>> >> Seja M uma matriz nxn com x na diagonal principal, e a>0 nas demais
>> >> posições. Calcule det(M).
>> >>
>> >> Alguém poderia me indicar um caminho para seguir? Eu não consegui
>> avançar
>> >> nada nessa questão :(
>> >>
>> >> Att.
>> >>
>> >> Eduardo
>> >>
>> >> --
>> >> Esta mensagem foi verificada pelo sistema de antivírus e
>> >> acredita-se estar livre de perigo.
>>
>> --
>> Esta mensagem foi verificada pelo sistema de antivírus e
>>  acredita-se estar livre de perigo.
>>
>>
>> =
>> Instru�ões para entrar na lista, sair da lista e usar a lista em
>> http://www.mat.puc-rio.br/~obmlistas/obm-l.html
>> =
>>
>
>
> --
> Esta mensagem foi verificada pelo sistema de antivírus e
> acredita-se estar livre de perigo.
>

-- 
Esta mensagem foi verificada pelo sistema de antiv�rus e
 acredita-se estar livre de perigo.



[obm-l] Re: [obm-l] Re: [obm-l] números especiais OMERJ 2015

2015-10-15 Por tôpico Matheus Secco
Um exemplo com quatro é 510, 511, 512, 513

2015-10-15 21:43 GMT-03:00 Bernardo Freitas Paulo da Costa <
bernardo...@gmail.com>:

> 2015-10-15 16:42 GMT-03:00 Luís :
> > Sauda,c~oes,
> >
> > Um número é dito especial se ele tem dois ou mais algarismos
> > e é múltiplo da soma dos seus algarismos. Por exemplo, 12 é
> > especial pois é múltiplo de 1 + 2 = 3.
> >
> > a) encontre três números especiais consecutivos;
>
> Não pensei em nada muito especial, mas se x = ab com dois dígitos
> ímpares, a soma é par.
> Logo, para 1b, 3b, 5b, 7b e 9b não precisa testar se os números são
> especiais ou não, pois é impossível haver três consecutivos, já que a
> cada dois um não é. Logo restam os números da forma 2b, 4b, 6b e 8b
> (com dois dígitos). Daí em diante um pouco de força bruta acha três
> consecutivos.
>
> > b) encontre quatro números especiais consecutivos.
>
> Ainda não achei estes. Acredito que tenha que usar divisibilidade por
> 3 e 4; eu não usei por 3 no caso anterior porque acabou sendo mais
> fácil a força bruta mesmo.
> --
> Bernardo Freitas Paulo da Costa
>
> --
> Esta mensagem foi verificada pelo sistema de antivírus e
>  acredita-se estar livre de perigo.
>
>
> =
> Instru�ões para entrar na lista, sair da lista e usar a lista em
> http://www.mat.puc-rio.br/~obmlistas/obm-l.html
> =
>

-- 
Esta mensagem foi verificada pelo sistema de antiv�rus e
 acredita-se estar livre de perigo.



Re: [obm-l] Re: Re: Sobre mdc de polinômios

2015-08-05 Por tôpico Matheus Secco
Se entendi bem o que você está perguntando, o Algoritmo de Euclides é uma 
maneira de se calcular o mdc de dois polinomios. 

Enviado do meu iPhone

 Em 05/08/2015, às 13:13, Listeiro 037 listeiro_...@yahoo.com.br escreveu:
 
 
 Olá Ralph. Agradeço pela resposta.
 
 Compreendi que se tratam de duas representações.
 
 Agora tenho mais uma dúvida: existiria algo semelhante a uma técnica no
 caso de se representar polinômios, por exemplo, por números, para saber
 se eles tem algo de notável, seja com um mdc ou co-primos? Talvez
 existam várias. É possível? Grato novamente.
 
 
 Em Wed, 5 Aug 2015 11:49:15 -0300
 Ralph Teixeira ralp...@gmail.com escreveu:
 
 Oi, Listeiro.
 
 A chave eh notar que divisibilidade de polinomios nao eh a mesma
 coisa que divisibilidade dos numeros que eles representam (quando
 voce substitui x). Digamos, exagerando, que nao tem nada a ver um com
 o outro.
 
 Um polinomio p(x) eh divisivel por um polinomio D(x) quando pode se
 escrever p(x)=q(x)D(x) onde q(x) eh um polinomio;
 Um numero inteiro p eh divisivel por um numero inteiro D quando pode
 se escrever p=qD onde q eh um numero inteiro.
 
 Parecidas, mas diferentes. Nos polinomios, nao ha nenhuma referencia a
 numeros inteiros. Por isso mesmo o m.d.c. entre dois polinomios nao
 eh o m.d.c. entre os numeros inteiros que eles possam representar.
 
 O seu exemplo eh otimo para ilustrar a diferenca. Outros exemplos:
 
 p(x)=x^2-2 eh divisivel por q(x)=x-raiz(2). Mas nem faz muito sentido
 perguntar se p(2)=2 eh divisivel por q(2)=2-raiz(2)...
 O polinomio p(x)=75 eh divisivel pelo polinomio q(x)=2, mas 75 nao eh
 divisivel por 2. Em geral, se p(x) e q(x) forem polinomios constantes
 quaisquer (nao-nulos), vale que p(x) eh divisivel por q(x), mas nao
 necessariamente esta divisibilidade se traduz aos NUMEROS que eles
 representam.
 
 Abraco, Ralph.
 
 2015-08-05 11:16 GMT-03:00 Listeiro 037 listeiro_...@yahoo.com.br:
 
 
 Saudações.
 
 Estive lendo e tentando resolver uns exercícios talvez mais básicos
 e encontrei uma dúvida, sobre uma coisa que causou uma confusão:
 mdc de polinômios.
 
 Por exemplo: x+1 e x-1. Eu sei que são irredutíveis, a única coisa
 em comum que divide ambos é 1.
 
 Mas mudando o contexto, posso dizer que eles são côngruos módulo 2,
 não é mesmo?
 
 E além disso para, por exemplo, x=23 temos x-1 - 23-1 = 22 e x+1 -
 23+1 - 24 e o mdc de 22 e 24 é 2. O que em determinado contexto
 contradiz o mdc=1.
 
 A confusão está feita. O que seriam cada uma destas três análises e
 quando são válidas e não-válidas? Quais os contextos?
 
 Desde já agradeço qualquer pista.
 
 --
 Esta mensagem foi verificada pelo sistema de antivírus e
 acredita-se estar livre de perigo.
 
 
 =
 Instru�ões para entrar na lista, sair da lista e usar a lista em
 http://www.mat.puc-rio.br/~obmlistas/obm-l.html
 =
 
 -- 
 Esta mensagem foi verificada pelo sistema de antivírus e
 acredita-se estar livre de perigo.
 
 
 =
 Instruções para entrar na lista, sair da lista e usar a lista em
 http://www.mat.puc-rio.br/~obmlistas/obm-l.html
 =

-- 
Esta mensagem foi verificada pelo sistema de antiv�rus e
 acredita-se estar livre de perigo.


=
Instru��es para entrar na lista, sair da lista e usar a lista em
http://www.mat.puc-rio.br/~obmlistas/obm-l.html
=


[obm-l] Re: [obm-l] Re: [obm-l] Re: [obm-l] Princípio das gavetas

2015-07-16 Por tôpico Matheus Secco
Respondendo a pergunta adicional que o Sávio propôs: se a é primo entre si
com n, qualquer conjunto com n-1 elementos, todos == a (mod m), mostra que
n é a melhor cota possível.
Sávio, você sabe dizer se estes são os únicos exemplos para n-1 elementos?
Abraços

2015-07-16 23:41 GMT-03:00 Sávio Ribas savio.ri...@gmail.com:

 Cheguei tarde e demorei a escrever, Secco! haha
 Abraços

 Em 16 de julho de 2015 22:33, Matheus Secco matheusse...@gmail.com
 escreveu:

 Sejam a_1, ..., a_n os números.
 Considere as somas a_1, a_1+a_2, a_1+a_2+a_3, ..., a_1+a_2+... + a_n.
 Se uma destas somas é divisível por n, o problema acaba.
 Caso contrário, pelo princípio da Casa dos Pombos, há duas somas que
 deixam o mesmo resto na divisão por n.
 Considerando a subtração destas duas somas, obtemos um subconjunto cuja
 soma dos elementos é divisível por n.

 Abraços,
 Matheus

 2015-07-16 23:23 GMT-03:00 marcone augusto araújo borges 
 marconeborge...@hotmail.com:

 Mostre que em qualquer coleção de n inteiros há um subconjunto cuja soma
 dos seus elementos é divisível por n

 --
 Esta mensagem foi verificada pelo sistema de antivírus e
 acredita-se estar livre de perigo.



 --
 Esta mensagem foi verificada pelo sistema de antivírus e
 acredita-se estar livre de perigo.



 --
 Esta mensagem foi verificada pelo sistema de antivírus e
 acredita-se estar livre de perigo.


-- 
Esta mensagem foi verificada pelo sistema de antiv�rus e
 acredita-se estar livre de perigo.



[obm-l] Re: [obm-l] Princípio das gavetas

2015-07-16 Por tôpico Matheus Secco
Sejam a_1, ..., a_n os números.
Considere as somas a_1, a_1+a_2, a_1+a_2+a_3, ..., a_1+a_2+... + a_n.
Se uma destas somas é divisível por n, o problema acaba.
Caso contrário, pelo princípio da Casa dos Pombos, há duas somas que deixam
o mesmo resto na divisão por n.
Considerando a subtração destas duas somas, obtemos um subconjunto cuja
soma dos elementos é divisível por n.

Abraços,
Matheus

2015-07-16 23:23 GMT-03:00 marcone augusto araújo borges 
marconeborge...@hotmail.com:

 Mostre que em qualquer coleção de n inteiros há um subconjunto cuja soma
 dos seus elementos é divisível por n

 --
 Esta mensagem foi verificada pelo sistema de antivírus e
 acredita-se estar livre de perigo.


-- 
Esta mensagem foi verificada pelo sistema de antiv�rus e
 acredita-se estar livre de perigo.



Re: [obm-l] Quadrados numa malha 10x10

2015-06-15 Por tôpico Matheus Secco
Oi gente!
Este problema é bem interessante.
Seus quadrados devem utilizar vértices da malha.
A grande graça é que os quadrados podem ser tortos.

Abraços

Enviado do meu iPhone

 Em 15/06/2015, às 11:29, Pacini Bores pacini.bo...@globo.com escreveu:
 
 Oi Douglas, desculpe, mas não entendi a pergunta.
 
 Um quadrado pode ser dividido em qualquer quantidade de quadrados( não 
 necessariamente congruentes) a partir de 4 e diferente de cinco.
 
 Tenho que utilizar inicialmente  somente os 100 quadradinhos ?
 
 Pacini
 
 Em 15 de junho de 2015 10:54, Douglas Oliveira de Lima 
 profdouglaso.del...@gmail.com escreveu:
 Olá, caros amigos, preciso de uma ajuda no seguinte problema:
 Quantos quadrados podemos formar numa malha 10x10? 
 
 Obs: Se souberem de algum artigo ou algum material escrito falando sobre o 
 assunto, ate mesmo esses livros de puzzles voltados para a matemática e 
 puderem me indicar , agradeço desde já.
 
 Um abraço do Douglas Oliveira
 
 -- 
 Esta mensagem foi verificada pelo sistema de antivírus e 
 acredita-se estar livre de perigo.
 
 
 -- 
 Esta mensagem foi verificada pelo sistema de antivírus e 
 acredita-se estar livre de perigo.

-- 
Esta mensagem foi verificada pelo sistema de antiv�rus e
 acredita-se estar livre de perigo.



[obm-l] Re: [obm-l] Combinatória

2015-06-14 Por tôpico Matheus Secco
Oi Marcone, associe um sinal de + a um livro escolhido e um sinal de - a um
livro não escolhido. Devemos colocar então 5 sinais de + e 7 sinais de -,
sem que haja dois sinais de + juntos.

Coloque os sinais de -`s primeiro. Eles gerarão 8 espaços e devemos colocar
no máximo um sinal de + nestes espaços. Basta assim escolher 5 dos 8
espaços criados, o que pode ser feito de C(8,5) = 56 maneiras.

Abraços,

Matheus Secco

2015-06-14 10:16 GMT-03:00 marcone augusto araújo borges 
marconeborge...@hotmail.com:

 Uma prateleira contém 12 livros. De quantas maneiras podemos escolher 5
 deles
 de modo que dois dos livros escolhidos nunca fiquem um ao lado do outro?



 --
 Esta mensagem foi verificada pelo sistema de antivírus e
 acredita-se estar livre de perigo.


-- 
Esta mensagem foi verificada pelo sistema de antiv�rus e
 acredita-se estar livre de perigo.



[obm-l] [obm-l] Ajuda em Questão da 3ª Fase da OBM Ano passa do

2007-10-08 Por tôpico Matheus Secco


Em um torneio de tênis de mesa (no qual nenhum jogo termina empatado), cada um 
dos n
participantes jogou uma única vez contra cada um dos outros. Sabe-se que, para 
todo k  2, não
existem k jogadores J1, J2, …, Jk tais que J1 ganhou de J2, J2 ganhou de J3, J3 
ganhou de J4, …, Jk – 1 ganhou de Jk, Jk ganhou de J1.
Prove que existe um jogador que ganhou de todos os outros e existe um jogador 
que perdeu de todos os outros.
_
Veja mapas e encontre as melhores rotas para fugir do trânsito com o Live 
Search Maps!
http://www.livemaps.com.br/index.aspx?tr=true

[obm-l]Para Nicolau e Ralph

2007-10-02 Por tôpico Matheus Secco

Nicolau Corção Saldanha e Ralph Teixeira,
É com prazer que vos informo que vocês foram alunos do meu tio (Professor 
Secco) no Impacto.
Não sei se ainda se lembram dele, mas fico muito contente de ter aqui na lista 
para ajudar-nos professores de vossos níveis, medalhistas de ouro na IMO.
Abraço e parabéns Nicolau pelo seu ótimo trabalho.
_
Conheça o Windows Live Spaces, a rede de relacionamentos conectada ao Messenger!
http://spaces.live.com/signup.aspx